JEE Questions for Physics Vectors Quiz 5 - MCQExams.com


Physics-Vectors-94536.png

  • Physics-Vectors-94537.png
  • 2)
    Physics-Vectors-94538.png

  • Physics-Vectors-94539.png

  • Physics-Vectors-94540.png

Physics-Vectors-94542.png
  • 2
  • 2)
    Physics-Vectors-94543.png

  • Physics-Vectors-94544.png

  • Physics-Vectors-94545.png
Five equal forces of 10 N each arc applied at one point and all are lying in one plane. If the angles between them are equal, the resultant force will be
  • Zero
  • 10 N
  • 20 N

  • Physics-Vectors-94547.png

Physics-Vectors-94548.png

  • Physics-Vectors-94549.png
  • 2)
    Physics-Vectors-94550.png

  • Physics-Vectors-94551.png

  • Physics-Vectors-94552.png
Any vector in an arbitrary direction can always be replaced by two (or three)
  • Parallel vectors which have the original vector as their resultant
  • Mutually perpendicular vectors which have the original vector as their resultant
  • Arbitrary vectors which have the original vector as their resultant
  • It is not possible to resolve a vector

Physics-Vectors-94554.png

  • Physics-Vectors-94555.png
  • 2)
    Physics-Vectors-94556.png

  • Physics-Vectors-94557.png

  • Physics-Vectors-94558.png

Physics-Vectors-94560.png
  • Directed towards the origin
  • Parallel to the position vector
  • Perpendicular to the position vector
  • Directed away from the origin
Which of the following is a scalar quantity?
  • Displacement
  • Electric field
  • Acceleration
  • Work

Physics-Vectors-94562.png
  • 1
  • 2)
    Physics-Vectors-94563.png

  • Physics-Vectors-94564.png

  • Physics-Vectors-94565.png
A boy walks uniformally along the sides of a rectangular park of size 400 m x 300 m, starting from one corner to the other corner diagonally opposite. Which of the following statement is incorrect?
  • He has travelled a distance of 700 m
  • His displacement is 700 m
  • His displacement is 500 m
  • His velocity is not uniform throughout the walk
Surface area is
  • Scalar
  • Vector
  • Neither scalar nor vector
  • Both scalar and vector

Physics-Vectors-94568.png

  • Physics-Vectors-94569.png
  • 2)
    Physics-Vectors-94570.png

  • Physics-Vectors-94571.png

  • Physics-Vectors-94572.png
There are two force vectors, one of 5 N and other of 12 N. At what angle the two vectors be added to get resultant vector of 17 N, 7 N and 13 N respectively.

  • Physics-Vectors-94574.png
  • 2)
    Physics-Vectors-94575.png

  • Physics-Vectors-94576.png

  • Physics-Vectors-94577.png

Physics-Vectors-94578.png
  • 0
  • 1
  • 2
  • 3
Can the resultant of 2 vectors he zero
  • Yes, when the 2 vectors are same in magnitude and direction
  • No
  • Yes, when the 2 vectors are same in magnitude but opposite in sense

  • Physics-Vectors-94580.png
The sum of the magnitudes of two forces acting at point is 18 and the magnitude of their resultant is 12. If the resultant is at 90o with the force of smaller magnitude, what are the magnitudes of forces?
  • 2, 5
  • 14, 4
  • 5, 13
  • 10, 8

Physics-Vectors-94582.png
  • 0, a, dθ
  • a, dθ, 0
  • 0, 0
  • None of these

Physics-Vectors-94584.png
  • 2R
  • 2)
    Physics-Vectors-94585.png

  • Physics-Vectors-94586.png

  • Physics-Vectors-94587.png

Physics-Vectors-94589.png

  • Physics-Vectors-94590.png
  • 2)
    Physics-Vectors-94591.png

  • Physics-Vectors-94592.png

  • Physics-Vectors-94593.png
The length of second\' s hand in watch is 1 cm. The change in velocity of its tip in 15 sec ⁡is
  • Zero
  • 2)
    Physics-Vectors-94594.png

  • Physics-Vectors-94595.png

  • Physics-Vectors-94596.png

Physics-Vectors-94598.png

  • Physics-Vectors-94599.png
  • 2)
    Physics-Vectors-94600.png

  • Physics-Vectors-94601.png

  • Physics-Vectors-94602.png

Physics-Vectors-94604.png

  • Physics-Vectors-94605.png
  • 2)
    Physics-Vectors-94606.png

  • Physics-Vectors-94607.png

  • Physics-Vectors-94608.png

  • Physics-Vectors-94609.png
A metal sphere is hung by a string fixed to a wall. The sphere is pushed away from the wall by a stick.The forces acting on the sphere are shown in the second diagram. Which of the following statements is wrong?
Physics-Vectors-94611.png

  • Physics-Vectors-94612.png
  • 2)
    Physics-Vectors-94613.png

  • Physics-Vectors-94614.png

  • Physics-Vectors-94615.png
A man crosses a 320 m wide river perpendicular to the current in 4 mm. If in still water he can swim with a speed 5/3 times that of the current, then the speed of the current, in m/min is
  • 60
  • 30
  • 40
  • 50

Physics-Vectors-94618.png
  • 90o
  • 180o
  • 120o
  • Zero

Physics-Vectors-94620.png

  • Physics-Vectors-94621.png
  • 2)
    Physics-Vectors-94622.png

  • Physics-Vectors-94623.png

  • Physics-Vectors-94624.png
A truck travelling due North at 20 m/s turns West and travels at the same speed. The change in its velocity be
  • 40 m/s N-W
  • 2)
    Physics-Vectors-94626.png
  • 40 m/s S-W

  • Physics-Vectors-94627.png
If the sum of two unit vectors is a unit vector, then magnitude of difference is

  • Physics-Vectors-94629.png
  • 2)
    Physics-Vectors-94630.png

  • Physics-Vectors-94631.png

  • Physics-Vectors-94632.png
An object of m kg with speed of v m/s strikes a wall at an angle θ and rebounds at the same speed and same angle. The magnitude of the change in momentum of the object will be
Physics-Vectors-94634.png

  • Physics-Vectors-94635.png
  • 2)
    Physics-Vectors-94636.png
  • 0

  • Physics-Vectors-94637.png
Two equal forces (P each) act at a point inclined to each other at an angle of 120o. The magnitude of their resultant is
  • P/2
  • P/4
  • P
  • 2P
0:0:1


Answered Not Answered Not Visited Correct : 0 Incorrect : 0

Practice Physics Quiz Questions and Answers